Clearance Items

  • Except that, as usual, there is no peer-reviewed manuscript providing evidence to support Axil's incoherent (but very "scientific" and intelligent sounding) babbling.


    Or go ahead Axil, please enlighten us all and prove me wrong by providing the peer-reviewed manuscript supporting your statement that the "cause of LENR is left handed chiral polarized magnetic flux tubes that instantly stabilize radioactive isotopes" supported by experiment.


    First look at this experiment


    External Content www.youtube.com
    Content embedded from external sources will not be displayed without your consent.
    Through the activation of external content, you agree that personal data may be transferred to third party platforms. We have provided more information on this in our privacy policy.

  • First look at this experiment


    External Content www.youtube.com
    Content embedded from external sources will not be displayed without your consent.
    Through the activation of external content, you agree that personal data may be transferred to third party platforms. We have provided more information on this in our privacy policy.


    OK, for the sake of discussion, I watched this video.


    Other's need not, as I can assure you, there is nothing in Bob Greenyer's video to support Axil's science-babble statement that "the cause of LENR is left handed chiral polarized magnetic flux tubes that instantly stabilize radioactive isotopes" supported by experiment.


    So Axil, please provide your defense for your statement.


    Responding by saying "first watch [Bob Greenyers video]" where he might or might not have identified an anomaly that others have published on (e.g. Alizadeh, Reinhardt) without any conclusions that necessitate invoking a non-standard model does not come remotely close to supporting your statement.


    This is because Axil can't support his statement.


    That is because, as usual, Axil's declarative statements are mere fictions of his own science-babble creation.


    As far as I can tell, this is because of some need to be perceived as knowledgeable and therefore significant. But I suppose there might be other motivations.


    Regardless, for those newbies who might think Axil actually has some expertise in this field, he has a near decade-long pattern of tossing out incoherent declarative "truths" that to a non-technical audience sound kind of impressive.

  • Nezt...


    How are the flux tubes produced


    The Surface Plasmon Polaritons generate two counter rotating spin currents that are chiral polarized. One current is left handed spin polarized and the other current is right handed spin polarized. These spin currents generate a falaco soliton. The spinning flux tubes connect these two rotating spin currents. Notice in the picture below that two counter rotating spin flows are represented.


    Pairs-of-Falaco-solitons-An-example-of-solid-2-dimensional-0-surgery.jpg


    I need to stop now because I will cause these posts to be sent to the clearance thread.


  • No, the only reason to stop is because you cannot support your statement "the cause of LENR is left handed chiral polarized magnetic flux tubes that instantly stabilize radioactive isotopes" supported by experiment.


    It's really quite simple: just provide the citation supporting your statement. I am confident that the moderators would be quite happy for you to provide any citations.


    In fact, if you can provide a supporting citation, they might even decide to promote a reasoned discussion to a new thread.


    But unfortunately, your history strongly suggests that you have no citation. Just more declarative science-babble.


  • Sorry, though these anomalies have been studied and published for several decades, none of your citations support your statement that "the cause of LENR is left handed chiral polarized magnetic flux tubes that instantly stabilize radioactive isotopes" supported by experiment.


    And in fact, no connection to LENR exists in your citations.


    Again, please provide a citation supporting your statement.


  • The citation supporting my statement was included in these threads


  • The citation supporting my statement was included in these threads

    Your two citations:


    1) Purcell effect was published in 1946. It explains nothing about LENR.

    2) Linton and Antiochos 2005 is a theoretical paper discussing the mathematical modeling of 3D flux and effects on subatomic particles.


    Neither paper has any published relationship to LENR. Neither paper has published relationship to each other (except that at a trivial level, they both involve electromagnetic radiation).

    Also, your doubling down by now claiming that "left-handed centrality" (which has no meaning, perhaps you meant to say "left-handed polarity") is "coupled to the weak force" is also not supported by your citations.


    You're attaining even lower levels of credibility.


    You provide no coherent explanation for how these relate to each other or to LENR. This is true despite your triumphant statements (without explanation) as to why (for example) these explain the "heart of LENR".


    And specifically, these two citations do not support your statement "the cause of LENR is left handed chiral polarized magnetic flux tubes that instantly stabilize radioactive isotopes supported by experiment." in any way.


    In other words, you are continuing your legacy of throwing "sciencey" sounding papers and sciencey sounding declarative non-sequiturs at the wall and hoping that they might stick.


    Instead, your flinging of unrelated-but-significant-sounding stuff against the wall merely keeps your credibility stuck at a near-zero level.


    In any case, it's certainly not helping the credibility of LENR in any way.

  • Also, since you so casually throw out statements like "the cause of LENR is left handed chiral polarized magnetic flux tubes", surely you can intelligently answer basic conceptual questions on the topic.


    For example:

    What are the two components of Spin Angular Momentum (SAM) in light?

    Which of these SAM components can be polarized right vs. left?

    What relevance does this have to plasmon polaritons?


    We all are waiting to gain new insight from you regarding your understanding of evanescent light waves.


  • You missed this reference


    https://www.nature.com/articles/524008b

    Particle physics: Only left-handed particles decay


    Read up on chirality. From what you are saying, I don't think you have seen this in your education.


    https://en.wikipedia.org/wiki/Chirality_(physics)


    Only left-handed fermions and right-handed antifermions interact with the weak interaction.




  • I'm sorry, but you're clearly the confused one here. The Purcell reference is applicable to light. (Light is not a sub-atomic particle and readily polarizes left or right). The second to 3D modeling of (electromagnetic) flux. Thowing in chirality of sub-atomic particles with effects describing light shows me that you have a great capacity to mix-metaphors.


    That's a no no in physics.


    But thank you. This does explain a lot about your confusion.


    (And I did not "miss" your Nature reference. You had not provided it).

  • I knew that I would be censored into the clearance thread and here we are.


    Yes, so now there's nothing stopping you from supporting your statement: "the cause of LENR is left handed chiral polarized magnetic flux tubes that instantly stabilize radioactive isotopes supported by experiment."


    Like with a publication. Remember to include a RELEVANT experiment (Greenyer is not, because there is no mention or claim to be relevant to "instantly stabilized left handed chiral polarized magnetic flux tubes".


    So far, your 0 for 3.

  • Oh, I just now see your Purcell effect thread. And I see that over there you also confused light with subatomic particle behavior.


    Wow. You are one confused individual!


    THHuxley responded to your original post correctly, in a much more patient and explicit manner than I did above.


    He also provided you some great links to help you understand this area better.


    BTW, we both have electrical engineering degrees and experience with particle physics and electro-optics, which probably explains the similar responses.


    You are really out of your element here, sorry to say.


    But try reading up. There is a lot of great material online these days.


    And please try to refrain from ignorant declarative statements claiming to provide new insight into LENR. Maybe after you have studied a bit you can furnish a written rationale for whatever new insights you think you might have come across.


  • A polariton is not a photon. The Polariton is a quasiparticle and if a quasiparticle has a spin then it is chiral. All spin is chiral.


  • I addressed THHuxley references. They all concerned light matter interaction. Light matter interactions boils down to polariton formation and behavior.


    You have given the laser based U232 stabilization experiment short shrift. Why?


    The Purcell effect

  • I addressed THHuxley references. They all concerned light matter interaction. Light matter interactions boils down to polariton formation and behavior.


    You responded to THH's references, but that does not mean you adequately addressed them.


    Here's the problem. Your references do not support your statements. Your brief assertions, even if they were true (and they are not) are not adequate to communicate an articulate theory.


    Asserting that you "addressed" these issues, or that you have a coherent explanation when you have nothing close to that, is not helping you or others.


    So my advice to you, if you want to learn anything or have a productive conversation, is to actually stick to relevant articles, or take the time to formulate your theories formally.


    Otherwise, you're really just spouting a bunch of hot air, as they say.

  • One simple example is explaining how the Purcell effect has relevance to subatomic particle behavior. You need to provide an explanation beyond just something you dreamed up, like an actual published article.


    Or if you really have a well-formulated theory for how the Purcell effect is relevant to subatomic particle decay, please publish it (or at least post it). Please include the appropriate differential equations describing your postulated relationships.


    That way, you won't just be a waste of time.


  • I am doing my best to layout an all encompassing LENR theory based on the SPP. I am not discouraged by the censorship that I am experiencing lately or the negativity from learned experts that could greatly help in perfecting the theory.

  • I am doing my best to layout an all encompassing LENR theory based on the SPP. I am not discouraged by the censorship that I am experiencing lately or the negativity from learned experts that could greatly help in perfecting the theory.


    Well, right off the bat you started running in the wrong direction. So I really can't help you until you decide to stop and turn around.


    BTW, you ignored my basic questions above.


    How about describing the involvement of orbital vs spin angular momentum (or both) in your theory?

Subscribe to our newsletter

It's sent once a month, you can unsubscribe at anytime!

View archive of previous newsletters

* indicates required

Your email address will be used to send you email newsletters only. See our Privacy Policy for more information.

Our Partners

Supporting researchers for over 20 years
Want to Advertise or Sponsor LENR Forum?
CLICK HERE to contact us.